www.vorhilfe.de
Vorhilfe

Kostenlose Kommunikationsplattform für gegenseitige Hilfestellungen.
Hallo Gast!einloggen | registrieren ]
Startseite · Forum · Wissen · Kurse · Mitglieder · Team · Impressum
Forenbaum
^ Forenbaum
Status Vorhilfe
  Status Geisteswiss.
    Status Erdkunde
    Status Geschichte
    Status Jura
    Status Musik/Kunst
    Status Pädagogik
    Status Philosophie
    Status Politik/Wirtschaft
    Status Psychologie
    Status Religion
    Status Sozialwissenschaften
  Status Informatik
    Status Schule
    Status Hochschule
    Status Info-Training
    Status Wettbewerbe
    Status Praxis
    Status Internes IR
  Status Ingenieurwiss.
    Status Bauingenieurwesen
    Status Elektrotechnik
    Status Maschinenbau
    Status Materialwissenschaft
    Status Regelungstechnik
    Status Signaltheorie
    Status Sonstiges
    Status Technik
  Status Mathe
    Status Schulmathe
    Status Hochschulmathe
    Status Mathe-Vorkurse
    Status Mathe-Software
  Status Naturwiss.
    Status Astronomie
    Status Biologie
    Status Chemie
    Status Geowissenschaften
    Status Medizin
    Status Physik
    Status Sport
  Status Sonstiges / Diverses
  Status Sprachen
    Status Deutsch
    Status Englisch
    Status Französisch
    Status Griechisch
    Status Latein
    Status Russisch
    Status Spanisch
    Status Vorkurse
    Status Sonstiges (Sprachen)
  Status Neuerdings
  Status Internes VH
    Status Café VH
    Status Verbesserungen
    Status Benutzerbetreuung
    Status Plenum
    Status Datenbank-Forum
    Status Test-Forum
    Status Fragwürdige Inhalte
    Status VH e.V.

Gezeigt werden alle Foren bis zur Tiefe 2

Navigation
 Startseite...
 Neuerdings beta neu
 Forum...
 vorwissen...
 vorkurse...
 Werkzeuge...
 Nachhilfevermittlung beta...
 Online-Spiele beta
 Suchen
 Verein...
 Impressum
Das Projekt
Server und Internetanbindung werden durch Spenden finanziert.
Organisiert wird das Projekt von unserem Koordinatorenteam.
Hunderte Mitglieder helfen ehrenamtlich in unseren moderierten Foren.
Anbieter der Seite ist der gemeinnützige Verein "Vorhilfe.de e.V.".
Partnerseiten
Dt. Schulen im Ausland: Mathe-Seiten:

Open Source FunktionenplotterFunkyPlot: Kostenloser und quelloffener Funktionenplotter für Linux und andere Betriebssysteme
Forum "Maßtheorie" - Monotone Klasse
Monotone Klasse < Maßtheorie < Maß/Integrat-Theorie < Analysis < Hochschule < Mathe < Vorhilfe
Ansicht: [ geschachtelt ] | ^ Forum "Maßtheorie"  | ^^ Alle Foren  | ^ Forenbaum  | Materialien

Monotone Klasse: Tipp, Idee
Status: (Frage) beantwortet Status 
Datum: 10:45 Fr 01.05.2020
Autor: TS85

Aufgabe
Sei X eine nichtleere Menge und [mm] \epsilon \subset \mathcal{P}(X). [/mm] Sei weiter [mm] \mathcal{M}(\epsilon) [/mm] die von [mm] \Epsilon [/mm] erzeugte monotone Mengenfamilie. Zeigen Sie, dass für jedes A [mm] \in \mathcal{M}(\epsilon) [/mm] eine Folge [mm] E_1,E_2,... [/mm] in [mm] \epsilon [/mm] existiert mit

A [mm] \subset \bigcup_{i=1}^{\infty}E_k. [/mm]

Hallo,

mein bisheriger Kenntnisstand:

[mm] \mathcal{R} \subset [/mm] P(X), [mm] \mathcal{R} \not= \emptyset [/mm]
heißt monotone Mengenfamilie, falls [mm] \mathcal{R} [/mm] die
folgenden Eigenschaften hat:
[mm] A_1 \subset A_2 \subset A_3 \subset [/mm] ... und jedes [mm] A_i \in \mathcal{R}, [/mm] dann ist auch [mm] \bigcup_{i=1}^{\infty} \in \mathcal{R} [/mm] (umgekehrt die Obermengen mit dem Schnitt).

[mm] \mathcal{M}(\epsilon) [/mm] ist die kleinste mon. Mengenfamilie in P(X), die [mm] \epsilon [/mm] enthält.
Sind alle [mm] \mathcal{R}_i [/mm] monot. Familien, so ist auch
[mm] \bigcap_{\mathcal{R} ist monoton \subset P(X) mit \epsilon \subset \mathcal{R}}^{\mathcal{R}_i} [/mm] eine monotone Familie.

Sei z.B. [mm] E_n=\{1,2,...,n\}, \epsilon=\{E_1,E_2,...\}. [/mm]
Dann gilt [mm] \mathcal{M}(\epsilon)=\{E_1,E_2,...,\IN\} [/mm] (gilt nach Skript)

Ich habe bei dieser Aufgabe bisher überhaupt keinen Kenntnisstand, um was für einen Beweis es sich hier handeln könnte. Hat der Beweis etwas mit Abzählbarkeit zu tun?
Mir ist auch unklar, was genau monotone Familien sonst für Regeln besitzen. Im Skript folgt der Aufbau mit Mengenring, Sigmaring, Mengenalgebra, Sigmaalgebra hierarchisch, bevor später monotone Familien als 5. in Bezug auf "die Schnitte der jeweiligen Art ist wieder diese Art (z.B. Schnitt von Mengenringen ist Mengenring)" eingeführt wird.

Da hier die Vereinigung aller [mm] E_i [/mm] verwendet wird, muss vermutlich eine monoton wachsende Mengenfamilie vorliegen.
Wie kommt die Unterscheidung zustande, dass [mm] \mathcal{M}(\epsilon) [/mm] zusätzlich die natürlichen Zahlen (gegenüber dem Erzeuger [mm] \epsilon) [/mm] enthält? Dies impliziert ja eigentlich eher, dass
A [mm] \supset \bigcup_{i=1}^{\infty}E_k. [/mm]
gilt (da ich hier [mm] \IN=A [/mm] auswählen könnte)

Anregungen/Lösungsansätze wären dementsprechend hilfreich.

        
Bezug
Monotone Klasse: Mitteilung
Status: (Mitteilung) Reaktion unnötig Status 
Datum: 14:51 Fr 01.05.2020
Autor: Gonozal_IX

Hiho,

auch wenn mir noch keine Idee für einen Beweis eingefallen ist, mal ein paar Anmerkungen zu deiner Frage:

>  Sind alle [mm]\mathcal{R}_i[/mm] monot. Familien, so ist auch
> [mm]\bigcap_{\mathcal{R} ist monoton \subset P(X) mit \epsilon \subset \mathcal{R}}^{\mathcal{R}_i}[/mm] eine monotone Familie.

Dies entspricht der Definition eines []Hüllenoperators
D.h.  $ [mm] \mathcal{M}(\epsilon) [/mm] $ ist einfach die kleinste monotone Mengenfamilie, die [mm] \epsilon [/mm] enthält.

> Sei z.B. [mm]E_n=\{1,2,...,n\}, \epsilon=\{E_1,E_2,...\}.[/mm]
>  Dann gilt [mm]\mathcal{M}(\epsilon)=\{E_1,E_2,...,\IN\}[/mm] (gilt nach Skript)

Du solltest dir auch klar machen, warum das gilt.
Das Problem ist aber, dass [mm] $\epsilon$ [/mm] beliebig kompliziert sein kann und man [mm] $\mathcal{M}(\epsilon)$ [/mm] nicht immer so hinschreiben kann.

> Hat der Beweis etwas mit Abzählbarkeit zu tun?

Das wird an sicherlich auf die eine oder andere Art benötigen.

> Mir ist auch unklar, was genau monotone Familien sonst für Regeln besitzen.

Keine weiteren als dass die abzählbare Vereinigung einer aufsteigenden Familie von Mengen enthalten ist als auch der abzählbare Schnitt einer fallenden.

>  Im Skript folgt der Aufbau mit Mengenring, Sigmaring, Mengenalgebra, Sigmaalgebra hierarchisch, bevor später monotone Familien als 5. in Bezug auf "die Schnitte der jeweiligen Art ist wieder diese Art (z.B. Schnitt von Mengenringen ist Mengenring)" eingeführt wird.

Das passiert sicher in Vorbereitung auf die Dynkin-Systeme, weil sich mit denen nachher Beweise stark vereinfachen lassen. (es reicht später Beweise auf durchschnittsstabilen Erzeugern zu zeigen als auf der gesamten [mm] $\sigma$-Algebra). [/mm]

Hilfreich ist dafür bestimmt []das Bild bei Wikipedia.

> Da hier die Vereinigung aller [mm]E_i[/mm] verwendet wird

Das steht nirgends, dass alle verwendet werden.
Der Beweis ist bspw. für alle Elemente im Erzeuger $E [mm] \in \epsilon$ [/mm] trivial, man nimmt einfach [mm] $E_i \equiv [/mm] E$ für alle $i [mm] \in \IN$ [/mm]

> Wie kommt die Unterscheidung zustande, dass [mm]\mathcal{M}(\epsilon)[/mm] zusätzlich die natürlichen Zahlen (gegenüber dem Erzeuger [mm]\epsilon)[/mm] enthält?

Das ist nur bei deinem Beispiel der Fall und stimmt im Allgemeinen gar nicht!!
Mach dir das klar, dass [mm] $\IN$ [/mm] bei deinem Beispiel die einzige Menge ist, die dazugenommen werden muss um eine monotone Klasse zu erhalten.

> Dies impliziert ja eigentlich eher, dass
> A [mm]\supset \bigcup_{i=1}^{\infty}E_k.[/mm]
> gilt (da ich hier [mm]\IN=A[/mm] auswählen könnte)

Das wäre offensichtlich sofort falsch…
Mit [mm] $\subset$ [/mm] ist bei der Aufgabe offensichtlich [mm] $\subseteq$ [/mm] gemeint (wird oft gleichbedeutend verwendet), andererseits wäre die Aussage offensichtlich falsch, wie dein Beispiel und die Wahl $A = [mm] \IN$ [/mm] sofort zeigt.

In deinem Beispiel gilt die Aussage für $A = [mm] \IN$ [/mm] offensichtlich, da dort ja gilt: $A = [mm] \IN \subset \IN [/mm] = [mm] \bigcup_{j=1}^\infty E_i$ [/mm]

> Anregungen/Lösungsansätze wären dementsprechend hilfreich.

Ich denke drüber nach…

Gruß,
Gono


Bezug
        
Bezug
Monotone Klasse: Antwort
Status: (Antwort) fertig Status 
Datum: 15:37 Fr 01.05.2020
Autor: tobit09

Hallo zusammen,


eine Teilmenge [mm] $A\subseteq [/mm] X$ möchte ich "gut" nennen, wenn sie die gewünschte Eigenschaft [mm] $\exists E_1,E_2,\ldots\in\mathcal{E}\colon A\subseteq\bigcup_{k=1}^\infty E_k$ [/mm] hat.

Zu zeigen ist somit, dass alle [mm] $A\in\mathcal{M}(\mathcal{E})$ [/mm] gut sind, mit anderen Worten dass [mm] $\mathcal{M}(\mathcal{E})\subseteq\mathcal{F}$ [/mm] für die Menge [mm] $\mathcal{F}$ [/mm] aller guten Teilmengen von $X$ gilt.

Dazu wiederum genügt es nach Definition von [mm] $\mathcal{M}(\mathcal{E})$ [/mm] zu zeigen, dass [mm] $\mathcal{F}$ [/mm] eine monotone Familie mit [mm] $\mathcal{F}\supseteq\mathcal{E}$ [/mm] ist.


Ist dieser Ansatz verständlich?

Kannst du [mm] $\mathcal{E}\subseteq\mathcal{F}$ [/mm] zeigen, also zeigen, dass alle [mm] $E\in\mathcal{E}$ [/mm] gut sind?

Kannst du zeigen, dass die Menge [mm] $\mathcal{F}$ [/mm] der guten Mengen eine monotone Familie ist, also die beiden definierenden Eigenschaften einer monotonen Familie nachrechnen?


Viele Grüße
Tobias

Bezug
                
Bezug
Monotone Klasse: Frage (beantwortet)
Status: (Frage) beantwortet Status 
Datum: 15:14 Sa 02.05.2020
Autor: TS85

Die Aufgabe ist mir ehrlich gesagt zu theoretisch und speziell, als dass ich sie vollkommen verstehen würde.

A [mm] \subseteq [/mm] X erfüllt die Anforderung, falls sie die Eigenschaft
[mm] \exists E_1,E_2,... \in \mathcal{E}: [/mm] A [mm] \subseteq \bigcup_{i=1}^{\infty}E_k [/mm]
besitzt. z.z. ist somit, dass [mm] \mathcal{E} \subseteq \mathcal{M}(\mathcal{E})\subseteq \mathcal{F} [/mm] für die Menge [mm] \mathcal{F} [/mm] aller anforderungserfüllenden Teilmengen von X. Damit dies gilt, muss [mm] \mathcal{F} [/mm] eine monotone Familie sein.

Da [mm] \mathcal{E}\subseteq \mathcal{P}(X) [/mm] gilt, muss mindestens eine monotone Familie vorhanden sein, welche [mm] \mathcal{E} [/mm] enthält.
Sei dazu [mm] \mathcal{M} [/mm] die Menge der monotonen Klassen, welche [mm] \mathcal{E} [/mm]
enthalten:

[mm] \mathcal{M}:=\{ \mathcal{M}' : \mathcal{E} \subseteq \mathcal{M}', \mathcal{M}' ist monotone Klasse\}. [/mm]
Nach der bereits genannten "Schnittmengenregel" gilt: [mm] \mathcal{F}=\bigcap \mathcal{M} [/mm] ist eine monotone Klasse.
Daraus folgt, dass [mm] \mathcal{E} \subseteq \mathcal{F} [/mm] gilt.

Sei nun [mm] \mathcal{M}' [/mm] die monotone Klasse, welche [mm] \mathcal{E} [/mm] enthält.
Dann gilt nach der Definition von [mm] \mathcal{M}: [/mm]

[mm] \mathcal{F} \subseteq \mathcal{M}' [/mm]

womit gezeigt ist, dass [mm] \mathcal{M}(\mathcal{E})\subseteq \mathcal{F}. [/mm]

Wie genau ich die Schnittmengen bzw. Vereinigungseigenschaft hier zeigen soll (mit aufsteigenden/absteigenden Mengenfolgen) ist mir unklar.
Verbesserungsvorschläge sind also gerne willkommen.


Bezug
                        
Bezug
Monotone Klasse: Antwort
Status: (Antwort) fertig Status 
Datum: 15:56 Sa 02.05.2020
Autor: tobit09


> A [mm]\subseteq[/mm] X erfüllt die Anforderung, falls sie die
> Eigenschaft
> [mm]\exists E_1,E_2,... \in \mathcal{E}:[/mm] A [mm]\subseteq \bigcup_{i=1}^{\infty}E_k[/mm]
>  
> besitzt.

Ja.

> z.z. ist somit, dass [mm]\mathcal{E} \subseteq \mathcal{M}(\mathcal{E})\subseteq \mathcal{F}[/mm]
> für die Menge [mm]\mathcal{F}[/mm] aller anforderungserfüllenden
> Teilmengen von X.

Ja, wobei [mm] $\mathcal{E}\subseteq\mathcal{M}(\mathcal{E})$ [/mm] ohnehin gilt und nicht mehr gezeigt werden braucht. Der entscheidende zu zeigende Punkt ist [mm] $\mathcal{M}(\mathcal{E})\subseteq\mathcal{F}$. [/mm]

> Damit dies gilt, muss [mm]\mathcal{F}[/mm] eine
> monotone Familie sein.

Jein. A priori könnte auch [mm] $\mathcal{M}(\mathcal{E})\subseteq\mathcal{F}$ [/mm] gelten, ohne dass [mm] $\mathcal{F}$ [/mm] eine monotone Familie wäre. Aber tatsächlich lässt sich zeigen, dass [mm] $\mathcal{F}$ [/mm] eine monotone Familie ist.


> Da [mm]\mathcal{E}\subseteq \mathcal{P}(X)[/mm] gilt, muss
> mindestens eine monotone Familie vorhanden sein, welche
> [mm]\mathcal{E}[/mm] enthält.

Z.B. [mm] $\mathcal{P}(X)$ [/mm] ist eine monotone Familie, die [mm] $\mathcal{E}$ [/mm] als Teilmenge hat.

>  Sei dazu [mm]\mathcal{M}[/mm] die Menge der monotonen Klassen,
> welche [mm]\mathcal{E}[/mm]
>  enthalten:
>  
> [mm]\mathcal{M}:=\{ \mathcal{M}' : \mathcal{E} \subseteq \mathcal{M}', \mathcal{M}' ist monotone Klasse\}.[/mm]
>  
> Nach der bereits genannten "Schnittmengenregel" gilt:
> [mm]\mathcal{F}=\bigcap \mathcal{M}[/mm] ist eine monotone Klasse.

Nach der "Schnittmengenregel" ist [mm] $\bigcap\mathcal{M}$ [/mm] eine monotone Klasse. Diese haben wir mit [mm] $\mathcal{M}(\mathcal{E})$ [/mm] bezeichnet. Es muss NICHT [mm] $\mathcal{F}=\mathcal{M}(\mathcal{E})$ [/mm] gelten; es kann durchaus [mm] $\mathcal{F}$ [/mm] eine echte Obermenge von [mm] $\mathcal{M}(\mathcal{E})$ [/mm] sein. Und dass immer [mm] $\mathcal{M}(\mathcal{E})\subseteq\mathcal{F}$ [/mm] gilt, müssen wir erst noch zeigen.

>  Daraus folgt, dass [mm]\mathcal{E} \subseteq \mathcal{F}[/mm]
> gilt.

Folgerichtig.


> Sei nun [mm]\mathcal{M}'[/mm] die monotone Klasse, welche
> [mm]\mathcal{E}[/mm] enthält.

Du meinst EINE monotone Klasse, die [mm] $\mathcal{E}$ [/mm] enthält? Im Allgemeinen gibt es viele monotone Klassen, die [mm] $\mathcal{E}$ [/mm] als Teilmenge haben.


>  Dann gilt nach der Definition von [mm]\mathcal{M}:[/mm]
>  
> [mm]\mathcal{F} \subseteq \mathcal{M}'[/mm]

Nein bzw. folgerichtig.


> womit gezeigt ist, dass [mm]\mathcal{M}(\mathcal{E})\subseteq \mathcal{F}.[/mm]

Nein.


> Wie genau ich die Schnittmengen bzw.
> Vereinigungseigenschaft hier zeigen soll (mit
> aufsteigenden/absteigenden Mengenfolgen) ist mir unklar.

Du musst natürlich irgendwo die Definition von [mm] $\mathcal{F}$ [/mm] ins Spiel bringen, wenn du Eigenschaften von [mm] $\mathcal{F}$ [/mm] zeigen möchtest.

Sei also z.B. [mm] $A_1,A_2,\ldots\in\mathcal{F}$ [/mm] eine Folge von Teilmengen von $X$ mit [mm] $A_1\subseteq A_2\subseteq\ldots$. [/mm]
Zu zeigen ist [mm] $\bigcup_{k=1}^\infty A_k\in\mathcal{F}$. [/mm]

Für jedes [mm] $k=1,2,\ldots$ [/mm] existieren nach Definition von [mm] $\mathcal{F}$ [/mm] wegen [mm] $A_k\in\mathcal{F}$ [/mm] Mengen [mm] $E_{1}^k, E_{2}^k,\ldots\in\mathcal{E}$ [/mm] mit [mm] $A_k\subseteq\bigcup_{n=1}^\infty E_n^k$. [/mm]

Um nun wie gewünscht [mm] $\bigcup_{k=1}^\infty A_k\in\mathcal{F}$ [/mm] zu zeigen, benötigen wir nach Definition von [mm] $\mathcal{F}$ [/mm] abzählbar viele Elemente von [mm] $\mathcal{E}$, [/mm] deren Vereinigung [mm] $\bigcup_{k=1}^\infty A_k$ [/mm] umfasst.

Wie wäre es damit, alle [mm] $E_n^k$ [/mm] für alle [mm] $n,k\in\IN$ [/mm] zusammenzuschmeißen? Sind das auch nur abzählbar viele?


>  Verbesserungsvorschläge sind also gerne willkommen.

Ich glaube, du hast schon verstanden, warum wir [mm] $\mathcal{M}(\mathcal{E})\subseteq\mathcal{F}$ [/mm] zeigen wollen.
Ich glaube, du hast noch nicht verstanden, wie wir dies tun wollen.

Meine Idee dazu:

Zeige:
1. [mm] $\mathcal{E}\subseteq\mathcal{F}$ [/mm] (d.h. [mm] $E\in\mathcal{F}$ [/mm] für alle [mm] $E\in\mathcal{E}$) [/mm]
2. [mm] $\mathcal{F}$ [/mm] ist eine monotone Familie.

Warum ist das nützlich?
Wenn du 1. und 2. gezeigt hast, hast du [mm] $\mathcal{F}\in\mathcal{M}$ [/mm] gezeigt.
Somit ist [mm] $\mathcal{M}(\varepsilon)=\bigcap\mathcal{M}\subseteq\mathcal{F}$: [/mm]
Denn jedes [mm] $A\in\bigcap\mathcal{M}$ [/mm] erfüllt [mm] $A\in\mathcal{M}'$ [/mm] für alle [mm] $\mathcal{M}'\in\mathcal{M}$, [/mm] insbesondere für [mm] $\mathcal{M}'=\mathcal{F}$. [/mm]

Gehe diese Argumentation bitte langsam Schritt für Schritt durch und frage dich bei jedem Schritt, ob er dir klar ist, oder du eine Rückfrage hast.


Viel Erfolg! :-)

Bezug
                                
Bezug
Monotone Klasse: Frage (beantwortet)
Status: (Frage) beantwortet Status 
Datum: 11:13 So 03.05.2020
Autor: TS85

Etwas unklar ist mir die Reihenfolge (der Antwort):
Dass zeigen von [mm] \bigcup_{i=k}^{\infty}A_k \in \mathcal{F} [/mm]
durch abzählbare Mengen
betrifft doch eigentlich bereits den unten genannten Punkt (zu führende Beweise)
2. [mm] \mathcal{F} [/mm] ist monotone Familie?
Unklar ist mir hier, wie [mm] \bigcap_{k=1}^{\infty}A_k [/mm] zu zeigen wäre,
da bisher von [mm] A_1\subseteq A_2 \subseteq [/mm] ... ausgegangen worden ist. Oder habe ich hier den Zusammenhang falsch verstanden?

Die Vereinigung [mm] \bigcup_{n=1}^{\infty}\bigcup_{k=1}^{\infty}E_n^k [/mm] umfasst diese abzählbar vielen Elemente von [mm] \mathcal{E}, [/mm] da [mm] E_n^k [/mm] abz. mit Indexmenge [mm] \IN [/mm]
und deswegen die Vereinigung wieder abz.?

Danke schonmal für die Hilfe, mir ist nun wenigstens der Grundaufbau für den Beweis klar geworden.

Bezug
                                        
Bezug
Monotone Klasse: Antwort
Status: (Antwort) fertig Status 
Datum: 18:07 So 03.05.2020
Autor: tobit09


> Etwas unklar ist mir die Reihenfolge (der Antwort):
>  Dass zeigen von [mm]\bigcup_{i=k}^{\infty}A_k \in \mathcal{F}[/mm]

Das soll vermutlich [mm] $\bigcup_{k=1}^\infty A_k\in\mathcal{F}$ [/mm] heißen.

> durch abzählbare Mengen
>  betrifft doch eigentlich bereits den unten genannten Punkt
> (zu führende Beweise)
> 2. [mm]\mathcal{F}[/mm] ist monotone Familie?

Genau.


>  Unklar ist mir hier, wie [mm]\bigcap_{k=1}^{\infty}A_k[/mm] zu
> zeigen wäre,
>  da bisher von [mm]A_1\subseteq A_2 \subseteq[/mm] ... ausgegangen
> worden ist. Oder habe ich hier den Zusammenhang falsch
> verstanden?

Zu zeigen sind für 2.:
a) Für alle [mm] $A_1\subseteq A_2\subseteq\ldots\in\mathcal{F}$ [/mm] gilt auch [mm] $\bigcup_{k=1}^\infty A_k\in\mathcal{F}$. [/mm]
b) Für alle [mm] $A_1\supseteq A_2\supseteq\ldots\in\mathcal{F}$ [/mm] gilt auch [mm] $\bigcap_{k=1}^\infty A_k\in\mathcal{F}$. [/mm]

Zu a) habe ich ja schon einiges geschrieben.
Zu b): Was bedeutet denn z.B. [mm] $A_1\in\mathcal{F}$? [/mm]
Was ist für [mm] $\bigcap_{k=1}^\infty A_k\in\mathcal{F}$ [/mm] zu zeigen?

Jetzt wieder zu a):

> Die Vereinigung
> [mm]\bigcup_{n=1}^{\infty}\bigcup_{k=1}^{\infty}E_n^k[/mm] umfasst
> diese abzählbar vielen Elemente von [mm]\mathcal{E},[/mm] da [mm]E_n^k[/mm]
> abz. mit Indexmenge [mm]\IN[/mm]
>  und deswegen die Vereinigung wieder abz.?

Weder [mm] $\mathcal{E}$, [/mm] noch irgendwelche der [mm] $E_n^k$ [/mm] müssen abzählbar sein. Aber die Menge [mm] $\IN\times\IN$ [/mm] ist bekanntlich (?) abzählbar und damit ist

[mm] $\bigcup_{n=1}^{\infty}\bigcup_{k=1}^{\infty}E_n^k=\bigcup_{(n,k)\in\IN\times\IN}E_n^k$ [/mm]

eine Darstellung einer Vereinigung abzählbar vieler Elemente von [mm] $\mathcal{E}$. [/mm]

Wegen [mm] $\bigcup_{k=1}^\infty A_k\subseteq \bigcup_{(n,k)\in\IN\times\IN}E_n^k$ [/mm] gilt somit wie gewünscht [mm] $\bigcup_{k=1}^\infty A_k\in\mathcal{F}$. [/mm]

Damit ist 2. a) gezeigt.

Bezug
                                                
Bezug
Monotone Klasse: Mitteilung
Status: (Mitteilung) Reaktion unnötig Status 
Datum: 09:52 Mo 04.05.2020
Autor: TS85

Ok Danke für die Hilfe, dass hat für mich vieles aufgeklärt.

Bezug
Ansicht: [ geschachtelt ] | ^ Forum "Maßtheorie"  | ^^ Alle Foren  | ^ Forenbaum  | Materialien


^ Seitenanfang ^
www.vorhilfe.de